$int_{0}^{infty} frac{e^{-x} sin(x)}{x} dx$ Evaluate Integral












25












$begingroup$


Compute the following integral:



$$int_{0}^{infty} frac{e^{-x} sin(x)}{x} dx$$



Any hint, suggestion is welcome.










share|cite|improve this question











$endgroup$












  • $begingroup$
    this looks like something that could be done with complex contour integration. en.wikipedia.org/wiki/Methods_of_contour_integration
    $endgroup$
    – Dennis Gulko
    Jun 18 '12 at 10:08










  • $begingroup$
    http://www.wolframalpha.com/input/?i=Integrate[%28e^-x*Sin[x]%2Fx%29%2C{x%2C0%2CInfinity} Ans = Pi/4=0.785398. In case you don't want to use limits and make it indefinite than use "Integrate[(e^-x*Sin[x]/x)]". This is not step by step solution obviously but may be helpful to you.
    $endgroup$
    – Rorschach
    Jun 18 '12 at 10:17








  • 7




    $begingroup$
    Consider $I(p)=int_{0}^{infty}frac{e^{-px}sin(x)}{x}dx$,then $frac{d}{dp}I(p)=-int_{0}^{infty}e^{-px}sin(x)dx$.$I(p)$ tends to zero when $p$ tends to infinity.Solve this differential equation and work out $I(1)$.That is the answer.
    $endgroup$
    – y zh
    Jun 18 '12 at 10:21












  • $begingroup$
    @yzhao Would you consider writing that as an answer?
    $endgroup$
    – anon
    Jun 18 '12 at 10:38
















25












$begingroup$


Compute the following integral:



$$int_{0}^{infty} frac{e^{-x} sin(x)}{x} dx$$



Any hint, suggestion is welcome.










share|cite|improve this question











$endgroup$












  • $begingroup$
    this looks like something that could be done with complex contour integration. en.wikipedia.org/wiki/Methods_of_contour_integration
    $endgroup$
    – Dennis Gulko
    Jun 18 '12 at 10:08










  • $begingroup$
    http://www.wolframalpha.com/input/?i=Integrate[%28e^-x*Sin[x]%2Fx%29%2C{x%2C0%2CInfinity} Ans = Pi/4=0.785398. In case you don't want to use limits and make it indefinite than use "Integrate[(e^-x*Sin[x]/x)]". This is not step by step solution obviously but may be helpful to you.
    $endgroup$
    – Rorschach
    Jun 18 '12 at 10:17








  • 7




    $begingroup$
    Consider $I(p)=int_{0}^{infty}frac{e^{-px}sin(x)}{x}dx$,then $frac{d}{dp}I(p)=-int_{0}^{infty}e^{-px}sin(x)dx$.$I(p)$ tends to zero when $p$ tends to infinity.Solve this differential equation and work out $I(1)$.That is the answer.
    $endgroup$
    – y zh
    Jun 18 '12 at 10:21












  • $begingroup$
    @yzhao Would you consider writing that as an answer?
    $endgroup$
    – anon
    Jun 18 '12 at 10:38














25












25








25


14



$begingroup$


Compute the following integral:



$$int_{0}^{infty} frac{e^{-x} sin(x)}{x} dx$$



Any hint, suggestion is welcome.










share|cite|improve this question











$endgroup$




Compute the following integral:



$$int_{0}^{infty} frac{e^{-x} sin(x)}{x} dx$$



Any hint, suggestion is welcome.







calculus integration limits improper-integrals






share|cite|improve this question















share|cite|improve this question













share|cite|improve this question




share|cite|improve this question








edited Jan 12 at 6:49









Abcd

3,02821235




3,02821235










asked Jun 18 '12 at 10:02









user 1357113user 1357113

22.4k877226




22.4k877226












  • $begingroup$
    this looks like something that could be done with complex contour integration. en.wikipedia.org/wiki/Methods_of_contour_integration
    $endgroup$
    – Dennis Gulko
    Jun 18 '12 at 10:08










  • $begingroup$
    http://www.wolframalpha.com/input/?i=Integrate[%28e^-x*Sin[x]%2Fx%29%2C{x%2C0%2CInfinity} Ans = Pi/4=0.785398. In case you don't want to use limits and make it indefinite than use "Integrate[(e^-x*Sin[x]/x)]". This is not step by step solution obviously but may be helpful to you.
    $endgroup$
    – Rorschach
    Jun 18 '12 at 10:17








  • 7




    $begingroup$
    Consider $I(p)=int_{0}^{infty}frac{e^{-px}sin(x)}{x}dx$,then $frac{d}{dp}I(p)=-int_{0}^{infty}e^{-px}sin(x)dx$.$I(p)$ tends to zero when $p$ tends to infinity.Solve this differential equation and work out $I(1)$.That is the answer.
    $endgroup$
    – y zh
    Jun 18 '12 at 10:21












  • $begingroup$
    @yzhao Would you consider writing that as an answer?
    $endgroup$
    – anon
    Jun 18 '12 at 10:38


















  • $begingroup$
    this looks like something that could be done with complex contour integration. en.wikipedia.org/wiki/Methods_of_contour_integration
    $endgroup$
    – Dennis Gulko
    Jun 18 '12 at 10:08










  • $begingroup$
    http://www.wolframalpha.com/input/?i=Integrate[%28e^-x*Sin[x]%2Fx%29%2C{x%2C0%2CInfinity} Ans = Pi/4=0.785398. In case you don't want to use limits and make it indefinite than use "Integrate[(e^-x*Sin[x]/x)]". This is not step by step solution obviously but may be helpful to you.
    $endgroup$
    – Rorschach
    Jun 18 '12 at 10:17








  • 7




    $begingroup$
    Consider $I(p)=int_{0}^{infty}frac{e^{-px}sin(x)}{x}dx$,then $frac{d}{dp}I(p)=-int_{0}^{infty}e^{-px}sin(x)dx$.$I(p)$ tends to zero when $p$ tends to infinity.Solve this differential equation and work out $I(1)$.That is the answer.
    $endgroup$
    – y zh
    Jun 18 '12 at 10:21












  • $begingroup$
    @yzhao Would you consider writing that as an answer?
    $endgroup$
    – anon
    Jun 18 '12 at 10:38
















$begingroup$
this looks like something that could be done with complex contour integration. en.wikipedia.org/wiki/Methods_of_contour_integration
$endgroup$
– Dennis Gulko
Jun 18 '12 at 10:08




$begingroup$
this looks like something that could be done with complex contour integration. en.wikipedia.org/wiki/Methods_of_contour_integration
$endgroup$
– Dennis Gulko
Jun 18 '12 at 10:08












$begingroup$
http://www.wolframalpha.com/input/?i=Integrate[%28e^-x*Sin[x]%2Fx%29%2C{x%2C0%2CInfinity} Ans = Pi/4=0.785398. In case you don't want to use limits and make it indefinite than use "Integrate[(e^-x*Sin[x]/x)]". This is not step by step solution obviously but may be helpful to you.
$endgroup$
– Rorschach
Jun 18 '12 at 10:17






$begingroup$
http://www.wolframalpha.com/input/?i=Integrate[%28e^-x*Sin[x]%2Fx%29%2C{x%2C0%2CInfinity} Ans = Pi/4=0.785398. In case you don't want to use limits and make it indefinite than use "Integrate[(e^-x*Sin[x]/x)]". This is not step by step solution obviously but may be helpful to you.
$endgroup$
– Rorschach
Jun 18 '12 at 10:17






7




7




$begingroup$
Consider $I(p)=int_{0}^{infty}frac{e^{-px}sin(x)}{x}dx$,then $frac{d}{dp}I(p)=-int_{0}^{infty}e^{-px}sin(x)dx$.$I(p)$ tends to zero when $p$ tends to infinity.Solve this differential equation and work out $I(1)$.That is the answer.
$endgroup$
– y zh
Jun 18 '12 at 10:21






$begingroup$
Consider $I(p)=int_{0}^{infty}frac{e^{-px}sin(x)}{x}dx$,then $frac{d}{dp}I(p)=-int_{0}^{infty}e^{-px}sin(x)dx$.$I(p)$ tends to zero when $p$ tends to infinity.Solve this differential equation and work out $I(1)$.That is the answer.
$endgroup$
– y zh
Jun 18 '12 at 10:21














$begingroup$
@yzhao Would you consider writing that as an answer?
$endgroup$
– anon
Jun 18 '12 at 10:38




$begingroup$
@yzhao Would you consider writing that as an answer?
$endgroup$
– anon
Jun 18 '12 at 10:38










8 Answers
8






active

oldest

votes


















43












$begingroup$

Yet a different approach: parametric integration. Let
$$
F(lambda)=int_{0}^{infty} frac{e^{-lambda x} sin(x)}{x},dx,qquadlambda>0.
$$
Then
$$
F'(lambda)=-int_{0}^{infty} e^{-lambda x} sin(x),dx=-frac{1}{1+lambda^2}.
$$
Integrating and taking into account that $lim_{lambdatoinfty}F(lambda)=0$ we have
$$
F(lambda)=fracpi2-arctanlambda
$$
and
$$
int_{0}^{infty} frac{e^{-x} sin(x)}{x},dx=F(1)=fracpi4.
$$






share|cite|improve this answer









$endgroup$













  • $begingroup$
    @Chris:I'm sorry for just noticing you comment,but Julian gives a detailed description of my idea.
    $endgroup$
    – y zh
    Jun 18 '12 at 11:21












  • $begingroup$
    Nice answer (+1). I enjoy seeing a variety of solutions!
    $endgroup$
    – robjohn
    Jun 18 '12 at 11:44



















19












$begingroup$

Using Laplace Transform,
$$mathcal{L}(sin(x)) = frac{1}{s^2 + 1}$$
$$mathcal{L}left(frac{sin(x)}{x}right) = int_r^infty frac{1}{s^2 + 1} ds = frac{pi}{2} - arctan(r)$$
Therefore,
$$int_0^infty e^{-rx} frac{sin(x)}{x} dx = frac{pi}{2} - arctan(r)$$
Substituting r = 1,
$$int_0^infty e^{-x} frac{sin(x)}{x} dx = frac{pi}{4}$$






share|cite|improve this answer











$endgroup$













  • $begingroup$
    thanks for your delicate solution!
    $endgroup$
    – user 1357113
    Jun 18 '12 at 10:57



















15












$begingroup$

Another approach:
$$begin{eqnarray*}
int_{0}^{infty} dx, frac{e^{-x} sin(x)}{x}
&=& int_{0}^{infty}dx, frac{e^{-x}}{x} sum_{k=0}^infty frac{(-1)^k x^{2k+1}}{(2k+1)!} \
&=& sum_{k=0}^infty frac{(-1)^k}{(2k+1)!}
int_{0}^{infty}dx, x^{2k} e^{-x} \
&=& sum_{k=0}^infty frac{(-1)^k}{(2k+1)!}(2k)! \
&=& sum_{k=0}^infty frac{(-1)^k}{2k+1}
hspace{5ex} textrm{(Leibniz series for $pi$)}\
&=& frac{pi}{4}.
end{eqnarray*}$$






share|cite|improve this answer









$endgroup$













  • $begingroup$
    this is another magic shot! Nice job! Thanks! :-)
    $endgroup$
    – user 1357113
    Jun 19 '12 at 5:52












  • $begingroup$
    @Chris: Thanks, Chris. Another good question!
    $endgroup$
    – user26872
    Jun 19 '12 at 7:40



















13












$begingroup$

Write this as
$$
lim_{epsilonto0}int_epsilon^{1/epsilon}frac{e^{-(1-i)x}-e^{-(1+i)x}}{2ix},mathrm{d}xtag{1}
$$
and then consider the path integral
$$
frac{1}{2i}int_{gamma_epsilon} e^{-z},frac{mathrm{d}z}{z}tag{2}
$$
where $gamma_epsilon$ comes in along the line $(1+i)x$, makes a quarter circle clockwise along $|z|=epsilon$, goes out along the line $(1-i)x$ and then back a quarter circle counter-clockwise along $|z|=1/epsilon$. There are no poles inside this path, so the integral in $(2)$ is $0$.



The part along $|z|=1/epsilon$ dies away exponentially as $epsilonto0$. The two parts along the lines sum to our integral, $(1)$, and the part along $|z|=epsilon$ tends to $frac14$ of the integral of $frac{1}{2iz}$ clockwise around the origin; that is, $-pi/4$. Since the sum of these parts is $0$, the limit in $(1)$ must be $pi/4$. That is,
$$
int_0^inftyfrac{e^{-x}sin(x)}{x}mathrm{d}x=frac{pi}{4}tag{3}
$$






share|cite|improve this answer









$endgroup$













  • $begingroup$
    @robjohn that's a nice solution! But I think that you should also justify that principal value of the integral equals integral itself.
    $endgroup$
    – qoqosz
    Jun 18 '12 at 11:08










  • $begingroup$
    @qoqosz: I am not using a principal value since $frac{sin(x)}{x}$ is a nice bounded function. I use the $epsilon$ and $1/epsilon$ to connect with the contour integral. Perhaps I am missing your concern.
    $endgroup$
    – robjohn
    Jun 18 '12 at 11:17










  • $begingroup$
    @robjohn $frac{x}{1+x^2}$ is also bounded but when integrating over $mathbb{R}$ it exists only in P.V. sense. Well, maybe it's not a 'nice' function ;).
    $endgroup$
    – qoqosz
    Jun 18 '12 at 11:21










  • $begingroup$
    @qoqosz: $e^{-x}frac{sin(x)}{x}$ is a nice bounded function that dies away exponentially. It converges absolutely over $[0,infty)$. In fact, all of the integrals above converge absolutely. What am I missing?
    $endgroup$
    – robjohn
    Jun 18 '12 at 11:31






  • 1




    $begingroup$
    @qoqosz: Ah, I just saw the improper-integral tag. In the Riemann integral sense, it is improper because the domain of integration is not bounded, but it is not improper in the Lebesgue sense.
    $endgroup$
    – robjohn
    Jun 18 '12 at 11:36





















9












$begingroup$

If you know a bit about Fourier theory. You could Parseval's theorem
$$int !dx ,f(x) g(x)^* = int !dxi,hat f(xi) hat g(xi)^* $$
with $f(x) = sin(x)/x$, $g(x) = Theta(x) e^{-x}$ and $hat{f}$, $hat{g}$ their Fourier transforms and $Theta(x)$ the Heaviside step function.



Hint: $hat{f}(xi) = tfrac12sqrt{frac{pi}{2}} [Theta(1-xi) + Theta(1+xi)] =sqrt{frac{pi}{2}} mathop{rm rect}(xi) $.






share|cite|improve this answer









$endgroup$













  • $begingroup$
    @Anon: Sorry for not defining: $Theta(x)$ is the Heaviside step function.
    $endgroup$
    – Fabian
    Jun 18 '12 at 10:24





















5












$begingroup$

Let



$$f(z) = frac{e^{-z+iz}}{z}$$



and let $C$ be the contour that travels along $0$ to $R$, makes a quarter of a circle around to $iR$ and back to $0$, properly indented around $0$ with a quarter circle of radius $delta$ to avoid the pole.



Contour



As $R to infty$, the integral over rounded part of the contour tends to $0$ and the part around $0$ tends to $-ifrac{pi}{2}$ (N.B. this is $-ifrac{pi}{2}$ of the residue at $z=0$) as $delta to 0$. Then by Cauchy's theorem:



$$
0=oint_C f(z),dz =\
int_0^infty frac{e^{-z+iz}}{z},dz -int_0^{infty} frac{e^{-iz-z}}{z},dz - ifrac{pi}{2}
$$



And upon taking imaginary parts and solving:



$$
frac{pi}{4}=int_0^infty frac{e^{-z}sin(z)}{z},dx
$$






share|cite|improve this answer











$endgroup$













  • $begingroup$
    hehe (+1). Glad to have the opportunity to learn more on complex analysis. :-)
    $endgroup$
    – user 1357113
    Jul 4 '13 at 16:06










  • $begingroup$
    @Chris'swisesister I thought you may be interested in a slightly different contour for variety
    $endgroup$
    – Argon
    Jul 4 '13 at 16:06










  • $begingroup$
    sure. I'm always interested in various ways. Thanks! :-)
    $endgroup$
    – user 1357113
    Jul 4 '13 at 16:07










  • $begingroup$
    Could you also add the contour (a picture) in case you have one ready?
    $endgroup$
    – user 1357113
    Jul 4 '13 at 16:09












  • $begingroup$
    @Chris'swisesister I could try and draw one, I will update this soon.
    $endgroup$
    – Argon
    Jul 4 '13 at 16:11



















3












$begingroup$

$$int_0^{infty} frac{e^{-x}sin x}{x},dx=int_0^{infty}int_0^{infty} e^{-x}sin x ,e^{-xy},dy,dx=int_0^{infty} int_0^{infty} e^{-x(1+y)}sin x,dx,dy$$
From integration by parts or otherwise, one can show that:
$$int_0^{infty}e^{-x(1+y)}sin x,dx=frac{1}{1+(1+y)^2}$$
Hence
$$int_0^{infty} int_0^{infty} e^{-x(1+y)}sin x,dx,dy=int_0^{infty} frac{dy}{1+(1+y)^2}=left(arctan(1+y)right|_0^{infty}=boxed{dfrac{pi}{4}}$$






share|cite|improve this answer









$endgroup$













  • $begingroup$
    Neat, but isn't that more or less the same as Julián Aguirre's answer?
    $endgroup$
    – IAmNoOne
    May 9 '14 at 0:12



















2












$begingroup$

$newcommand{+}{^{dagger}}%
newcommand{angles}[1]{leftlangle #1 rightrangle}%
newcommand{braces}[1]{leftlbrace #1 rightrbrace}%
newcommand{bracks}[1]{leftlbrack #1 rightrbrack}%
newcommand{ceil}[1]{,leftlceil #1 rightrceil,}%
newcommand{dd}{{rm d}}%
newcommand{down}{downarrow}%
newcommand{ds}[1]{displaystyle{#1}}%
newcommand{equalby}[1]{{#1 atop {= atop vphantom{huge A}}}}%
newcommand{expo}[1]{,{rm e}^{#1},}%
newcommand{fermi}{,{rm f}}%
newcommand{floor}[1]{,leftlfloor #1 rightrfloor,}%
newcommand{half}{{1 over 2}}%
newcommand{ic}{{rm i}}%
newcommand{iff}{Longleftrightarrow}
newcommand{imp}{Longrightarrow}%
newcommand{isdiv}{,left.rightvert,}%
newcommand{ket}[1]{leftvert #1rightrangle}%
newcommand{ol}[1]{overline{#1}}%
newcommand{pars}[1]{left( #1 right)}%
newcommand{partiald}[3]{frac{partial^{#1} #2}{partial #3^{#1}}}
newcommand{pp}{{cal P}}%
newcommand{root}[2]{,sqrt[#1]{,#2,},}%
newcommand{sech}{,{rm sech}}%
newcommand{sgn}{,{rm sgn}}%
newcommand{totald}[3]{frac{{rm d}^{#1} #2}{{rm d} #3^{#1}}}
newcommand{ul}[1]{underline{#1}}%
newcommand{verts}[1]{leftvert, #1 ,rightvert}$
begin{align}
&color{#00f}{largeint_{0}^{infty}{expo{-x}sinpars{x} over x},dd x}
=int_{0}^{infty}expo{-x}pars{halfint_{-1}^{1}expo{ic kx},dd k}
=halfint_{-1}^{1}int_{0}^{infty}expo{pars{ic k - 1}x},dd k
\[3mm]&=halfint_{-1}^{1}{1 over 1 - ic k},dd k
=int_{0}^{1}{dd k over 1 + k^{2}} = arctanpars{1}
=color{#00f}{Large{pi over 4}}
end{align}






share|cite|improve this answer











$endgroup$













    Your Answer





    StackExchange.ifUsing("editor", function () {
    return StackExchange.using("mathjaxEditing", function () {
    StackExchange.MarkdownEditor.creationCallbacks.add(function (editor, postfix) {
    StackExchange.mathjaxEditing.prepareWmdForMathJax(editor, postfix, [["$", "$"], ["\\(","\\)"]]);
    });
    });
    }, "mathjax-editing");

    StackExchange.ready(function() {
    var channelOptions = {
    tags: "".split(" "),
    id: "69"
    };
    initTagRenderer("".split(" "), "".split(" "), channelOptions);

    StackExchange.using("externalEditor", function() {
    // Have to fire editor after snippets, if snippets enabled
    if (StackExchange.settings.snippets.snippetsEnabled) {
    StackExchange.using("snippets", function() {
    createEditor();
    });
    }
    else {
    createEditor();
    }
    });

    function createEditor() {
    StackExchange.prepareEditor({
    heartbeatType: 'answer',
    autoActivateHeartbeat: false,
    convertImagesToLinks: true,
    noModals: true,
    showLowRepImageUploadWarning: true,
    reputationToPostImages: 10,
    bindNavPrevention: true,
    postfix: "",
    imageUploader: {
    brandingHtml: "Powered by u003ca class="icon-imgur-white" href="https://imgur.com/"u003eu003c/au003e",
    contentPolicyHtml: "User contributions licensed under u003ca href="https://creativecommons.org/licenses/by-sa/3.0/"u003ecc by-sa 3.0 with attribution requiredu003c/au003e u003ca href="https://stackoverflow.com/legal/content-policy"u003e(content policy)u003c/au003e",
    allowUrls: true
    },
    noCode: true, onDemand: true,
    discardSelector: ".discard-answer"
    ,immediatelyShowMarkdownHelp:true
    });


    }
    });














    draft saved

    draft discarded


















    StackExchange.ready(
    function () {
    StackExchange.openid.initPostLogin('.new-post-login', 'https%3a%2f%2fmath.stackexchange.com%2fquestions%2f159836%2fint-0-infty-frace-x-sinxx-dx-evaluate-integral%23new-answer', 'question_page');
    }
    );

    Post as a guest















    Required, but never shown

























    8 Answers
    8






    active

    oldest

    votes








    8 Answers
    8






    active

    oldest

    votes









    active

    oldest

    votes






    active

    oldest

    votes









    43












    $begingroup$

    Yet a different approach: parametric integration. Let
    $$
    F(lambda)=int_{0}^{infty} frac{e^{-lambda x} sin(x)}{x},dx,qquadlambda>0.
    $$
    Then
    $$
    F'(lambda)=-int_{0}^{infty} e^{-lambda x} sin(x),dx=-frac{1}{1+lambda^2}.
    $$
    Integrating and taking into account that $lim_{lambdatoinfty}F(lambda)=0$ we have
    $$
    F(lambda)=fracpi2-arctanlambda
    $$
    and
    $$
    int_{0}^{infty} frac{e^{-x} sin(x)}{x},dx=F(1)=fracpi4.
    $$






    share|cite|improve this answer









    $endgroup$













    • $begingroup$
      @Chris:I'm sorry for just noticing you comment,but Julian gives a detailed description of my idea.
      $endgroup$
      – y zh
      Jun 18 '12 at 11:21












    • $begingroup$
      Nice answer (+1). I enjoy seeing a variety of solutions!
      $endgroup$
      – robjohn
      Jun 18 '12 at 11:44
















    43












    $begingroup$

    Yet a different approach: parametric integration. Let
    $$
    F(lambda)=int_{0}^{infty} frac{e^{-lambda x} sin(x)}{x},dx,qquadlambda>0.
    $$
    Then
    $$
    F'(lambda)=-int_{0}^{infty} e^{-lambda x} sin(x),dx=-frac{1}{1+lambda^2}.
    $$
    Integrating and taking into account that $lim_{lambdatoinfty}F(lambda)=0$ we have
    $$
    F(lambda)=fracpi2-arctanlambda
    $$
    and
    $$
    int_{0}^{infty} frac{e^{-x} sin(x)}{x},dx=F(1)=fracpi4.
    $$






    share|cite|improve this answer









    $endgroup$













    • $begingroup$
      @Chris:I'm sorry for just noticing you comment,but Julian gives a detailed description of my idea.
      $endgroup$
      – y zh
      Jun 18 '12 at 11:21












    • $begingroup$
      Nice answer (+1). I enjoy seeing a variety of solutions!
      $endgroup$
      – robjohn
      Jun 18 '12 at 11:44














    43












    43








    43





    $begingroup$

    Yet a different approach: parametric integration. Let
    $$
    F(lambda)=int_{0}^{infty} frac{e^{-lambda x} sin(x)}{x},dx,qquadlambda>0.
    $$
    Then
    $$
    F'(lambda)=-int_{0}^{infty} e^{-lambda x} sin(x),dx=-frac{1}{1+lambda^2}.
    $$
    Integrating and taking into account that $lim_{lambdatoinfty}F(lambda)=0$ we have
    $$
    F(lambda)=fracpi2-arctanlambda
    $$
    and
    $$
    int_{0}^{infty} frac{e^{-x} sin(x)}{x},dx=F(1)=fracpi4.
    $$






    share|cite|improve this answer









    $endgroup$



    Yet a different approach: parametric integration. Let
    $$
    F(lambda)=int_{0}^{infty} frac{e^{-lambda x} sin(x)}{x},dx,qquadlambda>0.
    $$
    Then
    $$
    F'(lambda)=-int_{0}^{infty} e^{-lambda x} sin(x),dx=-frac{1}{1+lambda^2}.
    $$
    Integrating and taking into account that $lim_{lambdatoinfty}F(lambda)=0$ we have
    $$
    F(lambda)=fracpi2-arctanlambda
    $$
    and
    $$
    int_{0}^{infty} frac{e^{-x} sin(x)}{x},dx=F(1)=fracpi4.
    $$







    share|cite|improve this answer












    share|cite|improve this answer



    share|cite|improve this answer










    answered Jun 18 '12 at 11:07









    Julián AguirreJulián Aguirre

    68.2k24094




    68.2k24094












    • $begingroup$
      @Chris:I'm sorry for just noticing you comment,but Julian gives a detailed description of my idea.
      $endgroup$
      – y zh
      Jun 18 '12 at 11:21












    • $begingroup$
      Nice answer (+1). I enjoy seeing a variety of solutions!
      $endgroup$
      – robjohn
      Jun 18 '12 at 11:44


















    • $begingroup$
      @Chris:I'm sorry for just noticing you comment,but Julian gives a detailed description of my idea.
      $endgroup$
      – y zh
      Jun 18 '12 at 11:21












    • $begingroup$
      Nice answer (+1). I enjoy seeing a variety of solutions!
      $endgroup$
      – robjohn
      Jun 18 '12 at 11:44
















    $begingroup$
    @Chris:I'm sorry for just noticing you comment,but Julian gives a detailed description of my idea.
    $endgroup$
    – y zh
    Jun 18 '12 at 11:21






    $begingroup$
    @Chris:I'm sorry for just noticing you comment,but Julian gives a detailed description of my idea.
    $endgroup$
    – y zh
    Jun 18 '12 at 11:21














    $begingroup$
    Nice answer (+1). I enjoy seeing a variety of solutions!
    $endgroup$
    – robjohn
    Jun 18 '12 at 11:44




    $begingroup$
    Nice answer (+1). I enjoy seeing a variety of solutions!
    $endgroup$
    – robjohn
    Jun 18 '12 at 11:44











    19












    $begingroup$

    Using Laplace Transform,
    $$mathcal{L}(sin(x)) = frac{1}{s^2 + 1}$$
    $$mathcal{L}left(frac{sin(x)}{x}right) = int_r^infty frac{1}{s^2 + 1} ds = frac{pi}{2} - arctan(r)$$
    Therefore,
    $$int_0^infty e^{-rx} frac{sin(x)}{x} dx = frac{pi}{2} - arctan(r)$$
    Substituting r = 1,
    $$int_0^infty e^{-x} frac{sin(x)}{x} dx = frac{pi}{4}$$






    share|cite|improve this answer











    $endgroup$













    • $begingroup$
      thanks for your delicate solution!
      $endgroup$
      – user 1357113
      Jun 18 '12 at 10:57
















    19












    $begingroup$

    Using Laplace Transform,
    $$mathcal{L}(sin(x)) = frac{1}{s^2 + 1}$$
    $$mathcal{L}left(frac{sin(x)}{x}right) = int_r^infty frac{1}{s^2 + 1} ds = frac{pi}{2} - arctan(r)$$
    Therefore,
    $$int_0^infty e^{-rx} frac{sin(x)}{x} dx = frac{pi}{2} - arctan(r)$$
    Substituting r = 1,
    $$int_0^infty e^{-x} frac{sin(x)}{x} dx = frac{pi}{4}$$






    share|cite|improve this answer











    $endgroup$













    • $begingroup$
      thanks for your delicate solution!
      $endgroup$
      – user 1357113
      Jun 18 '12 at 10:57














    19












    19








    19





    $begingroup$

    Using Laplace Transform,
    $$mathcal{L}(sin(x)) = frac{1}{s^2 + 1}$$
    $$mathcal{L}left(frac{sin(x)}{x}right) = int_r^infty frac{1}{s^2 + 1} ds = frac{pi}{2} - arctan(r)$$
    Therefore,
    $$int_0^infty e^{-rx} frac{sin(x)}{x} dx = frac{pi}{2} - arctan(r)$$
    Substituting r = 1,
    $$int_0^infty e^{-x} frac{sin(x)}{x} dx = frac{pi}{4}$$






    share|cite|improve this answer











    $endgroup$



    Using Laplace Transform,
    $$mathcal{L}(sin(x)) = frac{1}{s^2 + 1}$$
    $$mathcal{L}left(frac{sin(x)}{x}right) = int_r^infty frac{1}{s^2 + 1} ds = frac{pi}{2} - arctan(r)$$
    Therefore,
    $$int_0^infty e^{-rx} frac{sin(x)}{x} dx = frac{pi}{2} - arctan(r)$$
    Substituting r = 1,
    $$int_0^infty e^{-x} frac{sin(x)}{x} dx = frac{pi}{4}$$







    share|cite|improve this answer














    share|cite|improve this answer



    share|cite|improve this answer








    edited Jun 18 '12 at 10:55

























    answered Jun 18 '12 at 10:41









    TenaliRamanTenaliRaman

    3,1641223




    3,1641223












    • $begingroup$
      thanks for your delicate solution!
      $endgroup$
      – user 1357113
      Jun 18 '12 at 10:57


















    • $begingroup$
      thanks for your delicate solution!
      $endgroup$
      – user 1357113
      Jun 18 '12 at 10:57
















    $begingroup$
    thanks for your delicate solution!
    $endgroup$
    – user 1357113
    Jun 18 '12 at 10:57




    $begingroup$
    thanks for your delicate solution!
    $endgroup$
    – user 1357113
    Jun 18 '12 at 10:57











    15












    $begingroup$

    Another approach:
    $$begin{eqnarray*}
    int_{0}^{infty} dx, frac{e^{-x} sin(x)}{x}
    &=& int_{0}^{infty}dx, frac{e^{-x}}{x} sum_{k=0}^infty frac{(-1)^k x^{2k+1}}{(2k+1)!} \
    &=& sum_{k=0}^infty frac{(-1)^k}{(2k+1)!}
    int_{0}^{infty}dx, x^{2k} e^{-x} \
    &=& sum_{k=0}^infty frac{(-1)^k}{(2k+1)!}(2k)! \
    &=& sum_{k=0}^infty frac{(-1)^k}{2k+1}
    hspace{5ex} textrm{(Leibniz series for $pi$)}\
    &=& frac{pi}{4}.
    end{eqnarray*}$$






    share|cite|improve this answer









    $endgroup$













    • $begingroup$
      this is another magic shot! Nice job! Thanks! :-)
      $endgroup$
      – user 1357113
      Jun 19 '12 at 5:52












    • $begingroup$
      @Chris: Thanks, Chris. Another good question!
      $endgroup$
      – user26872
      Jun 19 '12 at 7:40
















    15












    $begingroup$

    Another approach:
    $$begin{eqnarray*}
    int_{0}^{infty} dx, frac{e^{-x} sin(x)}{x}
    &=& int_{0}^{infty}dx, frac{e^{-x}}{x} sum_{k=0}^infty frac{(-1)^k x^{2k+1}}{(2k+1)!} \
    &=& sum_{k=0}^infty frac{(-1)^k}{(2k+1)!}
    int_{0}^{infty}dx, x^{2k} e^{-x} \
    &=& sum_{k=0}^infty frac{(-1)^k}{(2k+1)!}(2k)! \
    &=& sum_{k=0}^infty frac{(-1)^k}{2k+1}
    hspace{5ex} textrm{(Leibniz series for $pi$)}\
    &=& frac{pi}{4}.
    end{eqnarray*}$$






    share|cite|improve this answer









    $endgroup$













    • $begingroup$
      this is another magic shot! Nice job! Thanks! :-)
      $endgroup$
      – user 1357113
      Jun 19 '12 at 5:52












    • $begingroup$
      @Chris: Thanks, Chris. Another good question!
      $endgroup$
      – user26872
      Jun 19 '12 at 7:40














    15












    15








    15





    $begingroup$

    Another approach:
    $$begin{eqnarray*}
    int_{0}^{infty} dx, frac{e^{-x} sin(x)}{x}
    &=& int_{0}^{infty}dx, frac{e^{-x}}{x} sum_{k=0}^infty frac{(-1)^k x^{2k+1}}{(2k+1)!} \
    &=& sum_{k=0}^infty frac{(-1)^k}{(2k+1)!}
    int_{0}^{infty}dx, x^{2k} e^{-x} \
    &=& sum_{k=0}^infty frac{(-1)^k}{(2k+1)!}(2k)! \
    &=& sum_{k=0}^infty frac{(-1)^k}{2k+1}
    hspace{5ex} textrm{(Leibniz series for $pi$)}\
    &=& frac{pi}{4}.
    end{eqnarray*}$$






    share|cite|improve this answer









    $endgroup$



    Another approach:
    $$begin{eqnarray*}
    int_{0}^{infty} dx, frac{e^{-x} sin(x)}{x}
    &=& int_{0}^{infty}dx, frac{e^{-x}}{x} sum_{k=0}^infty frac{(-1)^k x^{2k+1}}{(2k+1)!} \
    &=& sum_{k=0}^infty frac{(-1)^k}{(2k+1)!}
    int_{0}^{infty}dx, x^{2k} e^{-x} \
    &=& sum_{k=0}^infty frac{(-1)^k}{(2k+1)!}(2k)! \
    &=& sum_{k=0}^infty frac{(-1)^k}{2k+1}
    hspace{5ex} textrm{(Leibniz series for $pi$)}\
    &=& frac{pi}{4}.
    end{eqnarray*}$$







    share|cite|improve this answer












    share|cite|improve this answer



    share|cite|improve this answer










    answered Jun 19 '12 at 0:13









    user26872user26872

    14.9k22773




    14.9k22773












    • $begingroup$
      this is another magic shot! Nice job! Thanks! :-)
      $endgroup$
      – user 1357113
      Jun 19 '12 at 5:52












    • $begingroup$
      @Chris: Thanks, Chris. Another good question!
      $endgroup$
      – user26872
      Jun 19 '12 at 7:40


















    • $begingroup$
      this is another magic shot! Nice job! Thanks! :-)
      $endgroup$
      – user 1357113
      Jun 19 '12 at 5:52












    • $begingroup$
      @Chris: Thanks, Chris. Another good question!
      $endgroup$
      – user26872
      Jun 19 '12 at 7:40
















    $begingroup$
    this is another magic shot! Nice job! Thanks! :-)
    $endgroup$
    – user 1357113
    Jun 19 '12 at 5:52






    $begingroup$
    this is another magic shot! Nice job! Thanks! :-)
    $endgroup$
    – user 1357113
    Jun 19 '12 at 5:52














    $begingroup$
    @Chris: Thanks, Chris. Another good question!
    $endgroup$
    – user26872
    Jun 19 '12 at 7:40




    $begingroup$
    @Chris: Thanks, Chris. Another good question!
    $endgroup$
    – user26872
    Jun 19 '12 at 7:40











    13












    $begingroup$

    Write this as
    $$
    lim_{epsilonto0}int_epsilon^{1/epsilon}frac{e^{-(1-i)x}-e^{-(1+i)x}}{2ix},mathrm{d}xtag{1}
    $$
    and then consider the path integral
    $$
    frac{1}{2i}int_{gamma_epsilon} e^{-z},frac{mathrm{d}z}{z}tag{2}
    $$
    where $gamma_epsilon$ comes in along the line $(1+i)x$, makes a quarter circle clockwise along $|z|=epsilon$, goes out along the line $(1-i)x$ and then back a quarter circle counter-clockwise along $|z|=1/epsilon$. There are no poles inside this path, so the integral in $(2)$ is $0$.



    The part along $|z|=1/epsilon$ dies away exponentially as $epsilonto0$. The two parts along the lines sum to our integral, $(1)$, and the part along $|z|=epsilon$ tends to $frac14$ of the integral of $frac{1}{2iz}$ clockwise around the origin; that is, $-pi/4$. Since the sum of these parts is $0$, the limit in $(1)$ must be $pi/4$. That is,
    $$
    int_0^inftyfrac{e^{-x}sin(x)}{x}mathrm{d}x=frac{pi}{4}tag{3}
    $$






    share|cite|improve this answer









    $endgroup$













    • $begingroup$
      @robjohn that's a nice solution! But I think that you should also justify that principal value of the integral equals integral itself.
      $endgroup$
      – qoqosz
      Jun 18 '12 at 11:08










    • $begingroup$
      @qoqosz: I am not using a principal value since $frac{sin(x)}{x}$ is a nice bounded function. I use the $epsilon$ and $1/epsilon$ to connect with the contour integral. Perhaps I am missing your concern.
      $endgroup$
      – robjohn
      Jun 18 '12 at 11:17










    • $begingroup$
      @robjohn $frac{x}{1+x^2}$ is also bounded but when integrating over $mathbb{R}$ it exists only in P.V. sense. Well, maybe it's not a 'nice' function ;).
      $endgroup$
      – qoqosz
      Jun 18 '12 at 11:21










    • $begingroup$
      @qoqosz: $e^{-x}frac{sin(x)}{x}$ is a nice bounded function that dies away exponentially. It converges absolutely over $[0,infty)$. In fact, all of the integrals above converge absolutely. What am I missing?
      $endgroup$
      – robjohn
      Jun 18 '12 at 11:31






    • 1




      $begingroup$
      @qoqosz: Ah, I just saw the improper-integral tag. In the Riemann integral sense, it is improper because the domain of integration is not bounded, but it is not improper in the Lebesgue sense.
      $endgroup$
      – robjohn
      Jun 18 '12 at 11:36


















    13












    $begingroup$

    Write this as
    $$
    lim_{epsilonto0}int_epsilon^{1/epsilon}frac{e^{-(1-i)x}-e^{-(1+i)x}}{2ix},mathrm{d}xtag{1}
    $$
    and then consider the path integral
    $$
    frac{1}{2i}int_{gamma_epsilon} e^{-z},frac{mathrm{d}z}{z}tag{2}
    $$
    where $gamma_epsilon$ comes in along the line $(1+i)x$, makes a quarter circle clockwise along $|z|=epsilon$, goes out along the line $(1-i)x$ and then back a quarter circle counter-clockwise along $|z|=1/epsilon$. There are no poles inside this path, so the integral in $(2)$ is $0$.



    The part along $|z|=1/epsilon$ dies away exponentially as $epsilonto0$. The two parts along the lines sum to our integral, $(1)$, and the part along $|z|=epsilon$ tends to $frac14$ of the integral of $frac{1}{2iz}$ clockwise around the origin; that is, $-pi/4$. Since the sum of these parts is $0$, the limit in $(1)$ must be $pi/4$. That is,
    $$
    int_0^inftyfrac{e^{-x}sin(x)}{x}mathrm{d}x=frac{pi}{4}tag{3}
    $$






    share|cite|improve this answer









    $endgroup$













    • $begingroup$
      @robjohn that's a nice solution! But I think that you should also justify that principal value of the integral equals integral itself.
      $endgroup$
      – qoqosz
      Jun 18 '12 at 11:08










    • $begingroup$
      @qoqosz: I am not using a principal value since $frac{sin(x)}{x}$ is a nice bounded function. I use the $epsilon$ and $1/epsilon$ to connect with the contour integral. Perhaps I am missing your concern.
      $endgroup$
      – robjohn
      Jun 18 '12 at 11:17










    • $begingroup$
      @robjohn $frac{x}{1+x^2}$ is also bounded but when integrating over $mathbb{R}$ it exists only in P.V. sense. Well, maybe it's not a 'nice' function ;).
      $endgroup$
      – qoqosz
      Jun 18 '12 at 11:21










    • $begingroup$
      @qoqosz: $e^{-x}frac{sin(x)}{x}$ is a nice bounded function that dies away exponentially. It converges absolutely over $[0,infty)$. In fact, all of the integrals above converge absolutely. What am I missing?
      $endgroup$
      – robjohn
      Jun 18 '12 at 11:31






    • 1




      $begingroup$
      @qoqosz: Ah, I just saw the improper-integral tag. In the Riemann integral sense, it is improper because the domain of integration is not bounded, but it is not improper in the Lebesgue sense.
      $endgroup$
      – robjohn
      Jun 18 '12 at 11:36
















    13












    13








    13





    $begingroup$

    Write this as
    $$
    lim_{epsilonto0}int_epsilon^{1/epsilon}frac{e^{-(1-i)x}-e^{-(1+i)x}}{2ix},mathrm{d}xtag{1}
    $$
    and then consider the path integral
    $$
    frac{1}{2i}int_{gamma_epsilon} e^{-z},frac{mathrm{d}z}{z}tag{2}
    $$
    where $gamma_epsilon$ comes in along the line $(1+i)x$, makes a quarter circle clockwise along $|z|=epsilon$, goes out along the line $(1-i)x$ and then back a quarter circle counter-clockwise along $|z|=1/epsilon$. There are no poles inside this path, so the integral in $(2)$ is $0$.



    The part along $|z|=1/epsilon$ dies away exponentially as $epsilonto0$. The two parts along the lines sum to our integral, $(1)$, and the part along $|z|=epsilon$ tends to $frac14$ of the integral of $frac{1}{2iz}$ clockwise around the origin; that is, $-pi/4$. Since the sum of these parts is $0$, the limit in $(1)$ must be $pi/4$. That is,
    $$
    int_0^inftyfrac{e^{-x}sin(x)}{x}mathrm{d}x=frac{pi}{4}tag{3}
    $$






    share|cite|improve this answer









    $endgroup$



    Write this as
    $$
    lim_{epsilonto0}int_epsilon^{1/epsilon}frac{e^{-(1-i)x}-e^{-(1+i)x}}{2ix},mathrm{d}xtag{1}
    $$
    and then consider the path integral
    $$
    frac{1}{2i}int_{gamma_epsilon} e^{-z},frac{mathrm{d}z}{z}tag{2}
    $$
    where $gamma_epsilon$ comes in along the line $(1+i)x$, makes a quarter circle clockwise along $|z|=epsilon$, goes out along the line $(1-i)x$ and then back a quarter circle counter-clockwise along $|z|=1/epsilon$. There are no poles inside this path, so the integral in $(2)$ is $0$.



    The part along $|z|=1/epsilon$ dies away exponentially as $epsilonto0$. The two parts along the lines sum to our integral, $(1)$, and the part along $|z|=epsilon$ tends to $frac14$ of the integral of $frac{1}{2iz}$ clockwise around the origin; that is, $-pi/4$. Since the sum of these parts is $0$, the limit in $(1)$ must be $pi/4$. That is,
    $$
    int_0^inftyfrac{e^{-x}sin(x)}{x}mathrm{d}x=frac{pi}{4}tag{3}
    $$







    share|cite|improve this answer












    share|cite|improve this answer



    share|cite|improve this answer










    answered Jun 18 '12 at 10:45









    robjohnrobjohn

    266k27306630




    266k27306630












    • $begingroup$
      @robjohn that's a nice solution! But I think that you should also justify that principal value of the integral equals integral itself.
      $endgroup$
      – qoqosz
      Jun 18 '12 at 11:08










    • $begingroup$
      @qoqosz: I am not using a principal value since $frac{sin(x)}{x}$ is a nice bounded function. I use the $epsilon$ and $1/epsilon$ to connect with the contour integral. Perhaps I am missing your concern.
      $endgroup$
      – robjohn
      Jun 18 '12 at 11:17










    • $begingroup$
      @robjohn $frac{x}{1+x^2}$ is also bounded but when integrating over $mathbb{R}$ it exists only in P.V. sense. Well, maybe it's not a 'nice' function ;).
      $endgroup$
      – qoqosz
      Jun 18 '12 at 11:21










    • $begingroup$
      @qoqosz: $e^{-x}frac{sin(x)}{x}$ is a nice bounded function that dies away exponentially. It converges absolutely over $[0,infty)$. In fact, all of the integrals above converge absolutely. What am I missing?
      $endgroup$
      – robjohn
      Jun 18 '12 at 11:31






    • 1




      $begingroup$
      @qoqosz: Ah, I just saw the improper-integral tag. In the Riemann integral sense, it is improper because the domain of integration is not bounded, but it is not improper in the Lebesgue sense.
      $endgroup$
      – robjohn
      Jun 18 '12 at 11:36




















    • $begingroup$
      @robjohn that's a nice solution! But I think that you should also justify that principal value of the integral equals integral itself.
      $endgroup$
      – qoqosz
      Jun 18 '12 at 11:08










    • $begingroup$
      @qoqosz: I am not using a principal value since $frac{sin(x)}{x}$ is a nice bounded function. I use the $epsilon$ and $1/epsilon$ to connect with the contour integral. Perhaps I am missing your concern.
      $endgroup$
      – robjohn
      Jun 18 '12 at 11:17










    • $begingroup$
      @robjohn $frac{x}{1+x^2}$ is also bounded but when integrating over $mathbb{R}$ it exists only in P.V. sense. Well, maybe it's not a 'nice' function ;).
      $endgroup$
      – qoqosz
      Jun 18 '12 at 11:21










    • $begingroup$
      @qoqosz: $e^{-x}frac{sin(x)}{x}$ is a nice bounded function that dies away exponentially. It converges absolutely over $[0,infty)$. In fact, all of the integrals above converge absolutely. What am I missing?
      $endgroup$
      – robjohn
      Jun 18 '12 at 11:31






    • 1




      $begingroup$
      @qoqosz: Ah, I just saw the improper-integral tag. In the Riemann integral sense, it is improper because the domain of integration is not bounded, but it is not improper in the Lebesgue sense.
      $endgroup$
      – robjohn
      Jun 18 '12 at 11:36


















    $begingroup$
    @robjohn that's a nice solution! But I think that you should also justify that principal value of the integral equals integral itself.
    $endgroup$
    – qoqosz
    Jun 18 '12 at 11:08




    $begingroup$
    @robjohn that's a nice solution! But I think that you should also justify that principal value of the integral equals integral itself.
    $endgroup$
    – qoqosz
    Jun 18 '12 at 11:08












    $begingroup$
    @qoqosz: I am not using a principal value since $frac{sin(x)}{x}$ is a nice bounded function. I use the $epsilon$ and $1/epsilon$ to connect with the contour integral. Perhaps I am missing your concern.
    $endgroup$
    – robjohn
    Jun 18 '12 at 11:17




    $begingroup$
    @qoqosz: I am not using a principal value since $frac{sin(x)}{x}$ is a nice bounded function. I use the $epsilon$ and $1/epsilon$ to connect with the contour integral. Perhaps I am missing your concern.
    $endgroup$
    – robjohn
    Jun 18 '12 at 11:17












    $begingroup$
    @robjohn $frac{x}{1+x^2}$ is also bounded but when integrating over $mathbb{R}$ it exists only in P.V. sense. Well, maybe it's not a 'nice' function ;).
    $endgroup$
    – qoqosz
    Jun 18 '12 at 11:21




    $begingroup$
    @robjohn $frac{x}{1+x^2}$ is also bounded but when integrating over $mathbb{R}$ it exists only in P.V. sense. Well, maybe it's not a 'nice' function ;).
    $endgroup$
    – qoqosz
    Jun 18 '12 at 11:21












    $begingroup$
    @qoqosz: $e^{-x}frac{sin(x)}{x}$ is a nice bounded function that dies away exponentially. It converges absolutely over $[0,infty)$. In fact, all of the integrals above converge absolutely. What am I missing?
    $endgroup$
    – robjohn
    Jun 18 '12 at 11:31




    $begingroup$
    @qoqosz: $e^{-x}frac{sin(x)}{x}$ is a nice bounded function that dies away exponentially. It converges absolutely over $[0,infty)$. In fact, all of the integrals above converge absolutely. What am I missing?
    $endgroup$
    – robjohn
    Jun 18 '12 at 11:31




    1




    1




    $begingroup$
    @qoqosz: Ah, I just saw the improper-integral tag. In the Riemann integral sense, it is improper because the domain of integration is not bounded, but it is not improper in the Lebesgue sense.
    $endgroup$
    – robjohn
    Jun 18 '12 at 11:36






    $begingroup$
    @qoqosz: Ah, I just saw the improper-integral tag. In the Riemann integral sense, it is improper because the domain of integration is not bounded, but it is not improper in the Lebesgue sense.
    $endgroup$
    – robjohn
    Jun 18 '12 at 11:36













    9












    $begingroup$

    If you know a bit about Fourier theory. You could Parseval's theorem
    $$int !dx ,f(x) g(x)^* = int !dxi,hat f(xi) hat g(xi)^* $$
    with $f(x) = sin(x)/x$, $g(x) = Theta(x) e^{-x}$ and $hat{f}$, $hat{g}$ their Fourier transforms and $Theta(x)$ the Heaviside step function.



    Hint: $hat{f}(xi) = tfrac12sqrt{frac{pi}{2}} [Theta(1-xi) + Theta(1+xi)] =sqrt{frac{pi}{2}} mathop{rm rect}(xi) $.






    share|cite|improve this answer









    $endgroup$













    • $begingroup$
      @Anon: Sorry for not defining: $Theta(x)$ is the Heaviside step function.
      $endgroup$
      – Fabian
      Jun 18 '12 at 10:24


















    9












    $begingroup$

    If you know a bit about Fourier theory. You could Parseval's theorem
    $$int !dx ,f(x) g(x)^* = int !dxi,hat f(xi) hat g(xi)^* $$
    with $f(x) = sin(x)/x$, $g(x) = Theta(x) e^{-x}$ and $hat{f}$, $hat{g}$ their Fourier transforms and $Theta(x)$ the Heaviside step function.



    Hint: $hat{f}(xi) = tfrac12sqrt{frac{pi}{2}} [Theta(1-xi) + Theta(1+xi)] =sqrt{frac{pi}{2}} mathop{rm rect}(xi) $.






    share|cite|improve this answer









    $endgroup$













    • $begingroup$
      @Anon: Sorry for not defining: $Theta(x)$ is the Heaviside step function.
      $endgroup$
      – Fabian
      Jun 18 '12 at 10:24
















    9












    9








    9





    $begingroup$

    If you know a bit about Fourier theory. You could Parseval's theorem
    $$int !dx ,f(x) g(x)^* = int !dxi,hat f(xi) hat g(xi)^* $$
    with $f(x) = sin(x)/x$, $g(x) = Theta(x) e^{-x}$ and $hat{f}$, $hat{g}$ their Fourier transforms and $Theta(x)$ the Heaviside step function.



    Hint: $hat{f}(xi) = tfrac12sqrt{frac{pi}{2}} [Theta(1-xi) + Theta(1+xi)] =sqrt{frac{pi}{2}} mathop{rm rect}(xi) $.






    share|cite|improve this answer









    $endgroup$



    If you know a bit about Fourier theory. You could Parseval's theorem
    $$int !dx ,f(x) g(x)^* = int !dxi,hat f(xi) hat g(xi)^* $$
    with $f(x) = sin(x)/x$, $g(x) = Theta(x) e^{-x}$ and $hat{f}$, $hat{g}$ their Fourier transforms and $Theta(x)$ the Heaviside step function.



    Hint: $hat{f}(xi) = tfrac12sqrt{frac{pi}{2}} [Theta(1-xi) + Theta(1+xi)] =sqrt{frac{pi}{2}} mathop{rm rect}(xi) $.







    share|cite|improve this answer












    share|cite|improve this answer



    share|cite|improve this answer










    answered Jun 18 '12 at 10:20









    FabianFabian

    19.7k3674




    19.7k3674












    • $begingroup$
      @Anon: Sorry for not defining: $Theta(x)$ is the Heaviside step function.
      $endgroup$
      – Fabian
      Jun 18 '12 at 10:24




















    • $begingroup$
      @Anon: Sorry for not defining: $Theta(x)$ is the Heaviside step function.
      $endgroup$
      – Fabian
      Jun 18 '12 at 10:24


















    $begingroup$
    @Anon: Sorry for not defining: $Theta(x)$ is the Heaviside step function.
    $endgroup$
    – Fabian
    Jun 18 '12 at 10:24






    $begingroup$
    @Anon: Sorry for not defining: $Theta(x)$ is the Heaviside step function.
    $endgroup$
    – Fabian
    Jun 18 '12 at 10:24













    5












    $begingroup$

    Let



    $$f(z) = frac{e^{-z+iz}}{z}$$



    and let $C$ be the contour that travels along $0$ to $R$, makes a quarter of a circle around to $iR$ and back to $0$, properly indented around $0$ with a quarter circle of radius $delta$ to avoid the pole.



    Contour



    As $R to infty$, the integral over rounded part of the contour tends to $0$ and the part around $0$ tends to $-ifrac{pi}{2}$ (N.B. this is $-ifrac{pi}{2}$ of the residue at $z=0$) as $delta to 0$. Then by Cauchy's theorem:



    $$
    0=oint_C f(z),dz =\
    int_0^infty frac{e^{-z+iz}}{z},dz -int_0^{infty} frac{e^{-iz-z}}{z},dz - ifrac{pi}{2}
    $$



    And upon taking imaginary parts and solving:



    $$
    frac{pi}{4}=int_0^infty frac{e^{-z}sin(z)}{z},dx
    $$






    share|cite|improve this answer











    $endgroup$













    • $begingroup$
      hehe (+1). Glad to have the opportunity to learn more on complex analysis. :-)
      $endgroup$
      – user 1357113
      Jul 4 '13 at 16:06










    • $begingroup$
      @Chris'swisesister I thought you may be interested in a slightly different contour for variety
      $endgroup$
      – Argon
      Jul 4 '13 at 16:06










    • $begingroup$
      sure. I'm always interested in various ways. Thanks! :-)
      $endgroup$
      – user 1357113
      Jul 4 '13 at 16:07










    • $begingroup$
      Could you also add the contour (a picture) in case you have one ready?
      $endgroup$
      – user 1357113
      Jul 4 '13 at 16:09












    • $begingroup$
      @Chris'swisesister I could try and draw one, I will update this soon.
      $endgroup$
      – Argon
      Jul 4 '13 at 16:11
















    5












    $begingroup$

    Let



    $$f(z) = frac{e^{-z+iz}}{z}$$



    and let $C$ be the contour that travels along $0$ to $R$, makes a quarter of a circle around to $iR$ and back to $0$, properly indented around $0$ with a quarter circle of radius $delta$ to avoid the pole.



    Contour



    As $R to infty$, the integral over rounded part of the contour tends to $0$ and the part around $0$ tends to $-ifrac{pi}{2}$ (N.B. this is $-ifrac{pi}{2}$ of the residue at $z=0$) as $delta to 0$. Then by Cauchy's theorem:



    $$
    0=oint_C f(z),dz =\
    int_0^infty frac{e^{-z+iz}}{z},dz -int_0^{infty} frac{e^{-iz-z}}{z},dz - ifrac{pi}{2}
    $$



    And upon taking imaginary parts and solving:



    $$
    frac{pi}{4}=int_0^infty frac{e^{-z}sin(z)}{z},dx
    $$






    share|cite|improve this answer











    $endgroup$













    • $begingroup$
      hehe (+1). Glad to have the opportunity to learn more on complex analysis. :-)
      $endgroup$
      – user 1357113
      Jul 4 '13 at 16:06










    • $begingroup$
      @Chris'swisesister I thought you may be interested in a slightly different contour for variety
      $endgroup$
      – Argon
      Jul 4 '13 at 16:06










    • $begingroup$
      sure. I'm always interested in various ways. Thanks! :-)
      $endgroup$
      – user 1357113
      Jul 4 '13 at 16:07










    • $begingroup$
      Could you also add the contour (a picture) in case you have one ready?
      $endgroup$
      – user 1357113
      Jul 4 '13 at 16:09












    • $begingroup$
      @Chris'swisesister I could try and draw one, I will update this soon.
      $endgroup$
      – Argon
      Jul 4 '13 at 16:11














    5












    5








    5





    $begingroup$

    Let



    $$f(z) = frac{e^{-z+iz}}{z}$$



    and let $C$ be the contour that travels along $0$ to $R$, makes a quarter of a circle around to $iR$ and back to $0$, properly indented around $0$ with a quarter circle of radius $delta$ to avoid the pole.



    Contour



    As $R to infty$, the integral over rounded part of the contour tends to $0$ and the part around $0$ tends to $-ifrac{pi}{2}$ (N.B. this is $-ifrac{pi}{2}$ of the residue at $z=0$) as $delta to 0$. Then by Cauchy's theorem:



    $$
    0=oint_C f(z),dz =\
    int_0^infty frac{e^{-z+iz}}{z},dz -int_0^{infty} frac{e^{-iz-z}}{z},dz - ifrac{pi}{2}
    $$



    And upon taking imaginary parts and solving:



    $$
    frac{pi}{4}=int_0^infty frac{e^{-z}sin(z)}{z},dx
    $$






    share|cite|improve this answer











    $endgroup$



    Let



    $$f(z) = frac{e^{-z+iz}}{z}$$



    and let $C$ be the contour that travels along $0$ to $R$, makes a quarter of a circle around to $iR$ and back to $0$, properly indented around $0$ with a quarter circle of radius $delta$ to avoid the pole.



    Contour



    As $R to infty$, the integral over rounded part of the contour tends to $0$ and the part around $0$ tends to $-ifrac{pi}{2}$ (N.B. this is $-ifrac{pi}{2}$ of the residue at $z=0$) as $delta to 0$. Then by Cauchy's theorem:



    $$
    0=oint_C f(z),dz =\
    int_0^infty frac{e^{-z+iz}}{z},dz -int_0^{infty} frac{e^{-iz-z}}{z},dz - ifrac{pi}{2}
    $$



    And upon taking imaginary parts and solving:



    $$
    frac{pi}{4}=int_0^infty frac{e^{-z}sin(z)}{z},dx
    $$







    share|cite|improve this answer














    share|cite|improve this answer



    share|cite|improve this answer








    edited Jul 4 '13 at 16:40

























    answered Jul 4 '13 at 15:57









    ArgonArgon

    16.3k673122




    16.3k673122












    • $begingroup$
      hehe (+1). Glad to have the opportunity to learn more on complex analysis. :-)
      $endgroup$
      – user 1357113
      Jul 4 '13 at 16:06










    • $begingroup$
      @Chris'swisesister I thought you may be interested in a slightly different contour for variety
      $endgroup$
      – Argon
      Jul 4 '13 at 16:06










    • $begingroup$
      sure. I'm always interested in various ways. Thanks! :-)
      $endgroup$
      – user 1357113
      Jul 4 '13 at 16:07










    • $begingroup$
      Could you also add the contour (a picture) in case you have one ready?
      $endgroup$
      – user 1357113
      Jul 4 '13 at 16:09












    • $begingroup$
      @Chris'swisesister I could try and draw one, I will update this soon.
      $endgroup$
      – Argon
      Jul 4 '13 at 16:11


















    • $begingroup$
      hehe (+1). Glad to have the opportunity to learn more on complex analysis. :-)
      $endgroup$
      – user 1357113
      Jul 4 '13 at 16:06










    • $begingroup$
      @Chris'swisesister I thought you may be interested in a slightly different contour for variety
      $endgroup$
      – Argon
      Jul 4 '13 at 16:06










    • $begingroup$
      sure. I'm always interested in various ways. Thanks! :-)
      $endgroup$
      – user 1357113
      Jul 4 '13 at 16:07










    • $begingroup$
      Could you also add the contour (a picture) in case you have one ready?
      $endgroup$
      – user 1357113
      Jul 4 '13 at 16:09












    • $begingroup$
      @Chris'swisesister I could try and draw one, I will update this soon.
      $endgroup$
      – Argon
      Jul 4 '13 at 16:11
















    $begingroup$
    hehe (+1). Glad to have the opportunity to learn more on complex analysis. :-)
    $endgroup$
    – user 1357113
    Jul 4 '13 at 16:06




    $begingroup$
    hehe (+1). Glad to have the opportunity to learn more on complex analysis. :-)
    $endgroup$
    – user 1357113
    Jul 4 '13 at 16:06












    $begingroup$
    @Chris'swisesister I thought you may be interested in a slightly different contour for variety
    $endgroup$
    – Argon
    Jul 4 '13 at 16:06




    $begingroup$
    @Chris'swisesister I thought you may be interested in a slightly different contour for variety
    $endgroup$
    – Argon
    Jul 4 '13 at 16:06












    $begingroup$
    sure. I'm always interested in various ways. Thanks! :-)
    $endgroup$
    – user 1357113
    Jul 4 '13 at 16:07




    $begingroup$
    sure. I'm always interested in various ways. Thanks! :-)
    $endgroup$
    – user 1357113
    Jul 4 '13 at 16:07












    $begingroup$
    Could you also add the contour (a picture) in case you have one ready?
    $endgroup$
    – user 1357113
    Jul 4 '13 at 16:09






    $begingroup$
    Could you also add the contour (a picture) in case you have one ready?
    $endgroup$
    – user 1357113
    Jul 4 '13 at 16:09














    $begingroup$
    @Chris'swisesister I could try and draw one, I will update this soon.
    $endgroup$
    – Argon
    Jul 4 '13 at 16:11




    $begingroup$
    @Chris'swisesister I could try and draw one, I will update this soon.
    $endgroup$
    – Argon
    Jul 4 '13 at 16:11











    3












    $begingroup$

    $$int_0^{infty} frac{e^{-x}sin x}{x},dx=int_0^{infty}int_0^{infty} e^{-x}sin x ,e^{-xy},dy,dx=int_0^{infty} int_0^{infty} e^{-x(1+y)}sin x,dx,dy$$
    From integration by parts or otherwise, one can show that:
    $$int_0^{infty}e^{-x(1+y)}sin x,dx=frac{1}{1+(1+y)^2}$$
    Hence
    $$int_0^{infty} int_0^{infty} e^{-x(1+y)}sin x,dx,dy=int_0^{infty} frac{dy}{1+(1+y)^2}=left(arctan(1+y)right|_0^{infty}=boxed{dfrac{pi}{4}}$$






    share|cite|improve this answer









    $endgroup$













    • $begingroup$
      Neat, but isn't that more or less the same as Julián Aguirre's answer?
      $endgroup$
      – IAmNoOne
      May 9 '14 at 0:12
















    3












    $begingroup$

    $$int_0^{infty} frac{e^{-x}sin x}{x},dx=int_0^{infty}int_0^{infty} e^{-x}sin x ,e^{-xy},dy,dx=int_0^{infty} int_0^{infty} e^{-x(1+y)}sin x,dx,dy$$
    From integration by parts or otherwise, one can show that:
    $$int_0^{infty}e^{-x(1+y)}sin x,dx=frac{1}{1+(1+y)^2}$$
    Hence
    $$int_0^{infty} int_0^{infty} e^{-x(1+y)}sin x,dx,dy=int_0^{infty} frac{dy}{1+(1+y)^2}=left(arctan(1+y)right|_0^{infty}=boxed{dfrac{pi}{4}}$$






    share|cite|improve this answer









    $endgroup$













    • $begingroup$
      Neat, but isn't that more or less the same as Julián Aguirre's answer?
      $endgroup$
      – IAmNoOne
      May 9 '14 at 0:12














    3












    3








    3





    $begingroup$

    $$int_0^{infty} frac{e^{-x}sin x}{x},dx=int_0^{infty}int_0^{infty} e^{-x}sin x ,e^{-xy},dy,dx=int_0^{infty} int_0^{infty} e^{-x(1+y)}sin x,dx,dy$$
    From integration by parts or otherwise, one can show that:
    $$int_0^{infty}e^{-x(1+y)}sin x,dx=frac{1}{1+(1+y)^2}$$
    Hence
    $$int_0^{infty} int_0^{infty} e^{-x(1+y)}sin x,dx,dy=int_0^{infty} frac{dy}{1+(1+y)^2}=left(arctan(1+y)right|_0^{infty}=boxed{dfrac{pi}{4}}$$






    share|cite|improve this answer









    $endgroup$



    $$int_0^{infty} frac{e^{-x}sin x}{x},dx=int_0^{infty}int_0^{infty} e^{-x}sin x ,e^{-xy},dy,dx=int_0^{infty} int_0^{infty} e^{-x(1+y)}sin x,dx,dy$$
    From integration by parts or otherwise, one can show that:
    $$int_0^{infty}e^{-x(1+y)}sin x,dx=frac{1}{1+(1+y)^2}$$
    Hence
    $$int_0^{infty} int_0^{infty} e^{-x(1+y)}sin x,dx,dy=int_0^{infty} frac{dy}{1+(1+y)^2}=left(arctan(1+y)right|_0^{infty}=boxed{dfrac{pi}{4}}$$







    share|cite|improve this answer












    share|cite|improve this answer



    share|cite|improve this answer










    answered May 8 '14 at 15:32









    Pranav AroraPranav Arora

    8,5472562




    8,5472562












    • $begingroup$
      Neat, but isn't that more or less the same as Julián Aguirre's answer?
      $endgroup$
      – IAmNoOne
      May 9 '14 at 0:12


















    • $begingroup$
      Neat, but isn't that more or less the same as Julián Aguirre's answer?
      $endgroup$
      – IAmNoOne
      May 9 '14 at 0:12
















    $begingroup$
    Neat, but isn't that more or less the same as Julián Aguirre's answer?
    $endgroup$
    – IAmNoOne
    May 9 '14 at 0:12




    $begingroup$
    Neat, but isn't that more or less the same as Julián Aguirre's answer?
    $endgroup$
    – IAmNoOne
    May 9 '14 at 0:12











    2












    $begingroup$

    $newcommand{+}{^{dagger}}%
    newcommand{angles}[1]{leftlangle #1 rightrangle}%
    newcommand{braces}[1]{leftlbrace #1 rightrbrace}%
    newcommand{bracks}[1]{leftlbrack #1 rightrbrack}%
    newcommand{ceil}[1]{,leftlceil #1 rightrceil,}%
    newcommand{dd}{{rm d}}%
    newcommand{down}{downarrow}%
    newcommand{ds}[1]{displaystyle{#1}}%
    newcommand{equalby}[1]{{#1 atop {= atop vphantom{huge A}}}}%
    newcommand{expo}[1]{,{rm e}^{#1},}%
    newcommand{fermi}{,{rm f}}%
    newcommand{floor}[1]{,leftlfloor #1 rightrfloor,}%
    newcommand{half}{{1 over 2}}%
    newcommand{ic}{{rm i}}%
    newcommand{iff}{Longleftrightarrow}
    newcommand{imp}{Longrightarrow}%
    newcommand{isdiv}{,left.rightvert,}%
    newcommand{ket}[1]{leftvert #1rightrangle}%
    newcommand{ol}[1]{overline{#1}}%
    newcommand{pars}[1]{left( #1 right)}%
    newcommand{partiald}[3]{frac{partial^{#1} #2}{partial #3^{#1}}}
    newcommand{pp}{{cal P}}%
    newcommand{root}[2]{,sqrt[#1]{,#2,},}%
    newcommand{sech}{,{rm sech}}%
    newcommand{sgn}{,{rm sgn}}%
    newcommand{totald}[3]{frac{{rm d}^{#1} #2}{{rm d} #3^{#1}}}
    newcommand{ul}[1]{underline{#1}}%
    newcommand{verts}[1]{leftvert, #1 ,rightvert}$
    begin{align}
    &color{#00f}{largeint_{0}^{infty}{expo{-x}sinpars{x} over x},dd x}
    =int_{0}^{infty}expo{-x}pars{halfint_{-1}^{1}expo{ic kx},dd k}
    =halfint_{-1}^{1}int_{0}^{infty}expo{pars{ic k - 1}x},dd k
    \[3mm]&=halfint_{-1}^{1}{1 over 1 - ic k},dd k
    =int_{0}^{1}{dd k over 1 + k^{2}} = arctanpars{1}
    =color{#00f}{Large{pi over 4}}
    end{align}






    share|cite|improve this answer











    $endgroup$


















      2












      $begingroup$

      $newcommand{+}{^{dagger}}%
      newcommand{angles}[1]{leftlangle #1 rightrangle}%
      newcommand{braces}[1]{leftlbrace #1 rightrbrace}%
      newcommand{bracks}[1]{leftlbrack #1 rightrbrack}%
      newcommand{ceil}[1]{,leftlceil #1 rightrceil,}%
      newcommand{dd}{{rm d}}%
      newcommand{down}{downarrow}%
      newcommand{ds}[1]{displaystyle{#1}}%
      newcommand{equalby}[1]{{#1 atop {= atop vphantom{huge A}}}}%
      newcommand{expo}[1]{,{rm e}^{#1},}%
      newcommand{fermi}{,{rm f}}%
      newcommand{floor}[1]{,leftlfloor #1 rightrfloor,}%
      newcommand{half}{{1 over 2}}%
      newcommand{ic}{{rm i}}%
      newcommand{iff}{Longleftrightarrow}
      newcommand{imp}{Longrightarrow}%
      newcommand{isdiv}{,left.rightvert,}%
      newcommand{ket}[1]{leftvert #1rightrangle}%
      newcommand{ol}[1]{overline{#1}}%
      newcommand{pars}[1]{left( #1 right)}%
      newcommand{partiald}[3]{frac{partial^{#1} #2}{partial #3^{#1}}}
      newcommand{pp}{{cal P}}%
      newcommand{root}[2]{,sqrt[#1]{,#2,},}%
      newcommand{sech}{,{rm sech}}%
      newcommand{sgn}{,{rm sgn}}%
      newcommand{totald}[3]{frac{{rm d}^{#1} #2}{{rm d} #3^{#1}}}
      newcommand{ul}[1]{underline{#1}}%
      newcommand{verts}[1]{leftvert, #1 ,rightvert}$
      begin{align}
      &color{#00f}{largeint_{0}^{infty}{expo{-x}sinpars{x} over x},dd x}
      =int_{0}^{infty}expo{-x}pars{halfint_{-1}^{1}expo{ic kx},dd k}
      =halfint_{-1}^{1}int_{0}^{infty}expo{pars{ic k - 1}x},dd k
      \[3mm]&=halfint_{-1}^{1}{1 over 1 - ic k},dd k
      =int_{0}^{1}{dd k over 1 + k^{2}} = arctanpars{1}
      =color{#00f}{Large{pi over 4}}
      end{align}






      share|cite|improve this answer











      $endgroup$
















        2












        2








        2





        $begingroup$

        $newcommand{+}{^{dagger}}%
        newcommand{angles}[1]{leftlangle #1 rightrangle}%
        newcommand{braces}[1]{leftlbrace #1 rightrbrace}%
        newcommand{bracks}[1]{leftlbrack #1 rightrbrack}%
        newcommand{ceil}[1]{,leftlceil #1 rightrceil,}%
        newcommand{dd}{{rm d}}%
        newcommand{down}{downarrow}%
        newcommand{ds}[1]{displaystyle{#1}}%
        newcommand{equalby}[1]{{#1 atop {= atop vphantom{huge A}}}}%
        newcommand{expo}[1]{,{rm e}^{#1},}%
        newcommand{fermi}{,{rm f}}%
        newcommand{floor}[1]{,leftlfloor #1 rightrfloor,}%
        newcommand{half}{{1 over 2}}%
        newcommand{ic}{{rm i}}%
        newcommand{iff}{Longleftrightarrow}
        newcommand{imp}{Longrightarrow}%
        newcommand{isdiv}{,left.rightvert,}%
        newcommand{ket}[1]{leftvert #1rightrangle}%
        newcommand{ol}[1]{overline{#1}}%
        newcommand{pars}[1]{left( #1 right)}%
        newcommand{partiald}[3]{frac{partial^{#1} #2}{partial #3^{#1}}}
        newcommand{pp}{{cal P}}%
        newcommand{root}[2]{,sqrt[#1]{,#2,},}%
        newcommand{sech}{,{rm sech}}%
        newcommand{sgn}{,{rm sgn}}%
        newcommand{totald}[3]{frac{{rm d}^{#1} #2}{{rm d} #3^{#1}}}
        newcommand{ul}[1]{underline{#1}}%
        newcommand{verts}[1]{leftvert, #1 ,rightvert}$
        begin{align}
        &color{#00f}{largeint_{0}^{infty}{expo{-x}sinpars{x} over x},dd x}
        =int_{0}^{infty}expo{-x}pars{halfint_{-1}^{1}expo{ic kx},dd k}
        =halfint_{-1}^{1}int_{0}^{infty}expo{pars{ic k - 1}x},dd k
        \[3mm]&=halfint_{-1}^{1}{1 over 1 - ic k},dd k
        =int_{0}^{1}{dd k over 1 + k^{2}} = arctanpars{1}
        =color{#00f}{Large{pi over 4}}
        end{align}






        share|cite|improve this answer











        $endgroup$



        $newcommand{+}{^{dagger}}%
        newcommand{angles}[1]{leftlangle #1 rightrangle}%
        newcommand{braces}[1]{leftlbrace #1 rightrbrace}%
        newcommand{bracks}[1]{leftlbrack #1 rightrbrack}%
        newcommand{ceil}[1]{,leftlceil #1 rightrceil,}%
        newcommand{dd}{{rm d}}%
        newcommand{down}{downarrow}%
        newcommand{ds}[1]{displaystyle{#1}}%
        newcommand{equalby}[1]{{#1 atop {= atop vphantom{huge A}}}}%
        newcommand{expo}[1]{,{rm e}^{#1},}%
        newcommand{fermi}{,{rm f}}%
        newcommand{floor}[1]{,leftlfloor #1 rightrfloor,}%
        newcommand{half}{{1 over 2}}%
        newcommand{ic}{{rm i}}%
        newcommand{iff}{Longleftrightarrow}
        newcommand{imp}{Longrightarrow}%
        newcommand{isdiv}{,left.rightvert,}%
        newcommand{ket}[1]{leftvert #1rightrangle}%
        newcommand{ol}[1]{overline{#1}}%
        newcommand{pars}[1]{left( #1 right)}%
        newcommand{partiald}[3]{frac{partial^{#1} #2}{partial #3^{#1}}}
        newcommand{pp}{{cal P}}%
        newcommand{root}[2]{,sqrt[#1]{,#2,},}%
        newcommand{sech}{,{rm sech}}%
        newcommand{sgn}{,{rm sgn}}%
        newcommand{totald}[3]{frac{{rm d}^{#1} #2}{{rm d} #3^{#1}}}
        newcommand{ul}[1]{underline{#1}}%
        newcommand{verts}[1]{leftvert, #1 ,rightvert}$
        begin{align}
        &color{#00f}{largeint_{0}^{infty}{expo{-x}sinpars{x} over x},dd x}
        =int_{0}^{infty}expo{-x}pars{halfint_{-1}^{1}expo{ic kx},dd k}
        =halfint_{-1}^{1}int_{0}^{infty}expo{pars{ic k - 1}x},dd k
        \[3mm]&=halfint_{-1}^{1}{1 over 1 - ic k},dd k
        =int_{0}^{1}{dd k over 1 + k^{2}} = arctanpars{1}
        =color{#00f}{Large{pi over 4}}
        end{align}







        share|cite|improve this answer














        share|cite|improve this answer



        share|cite|improve this answer








        edited May 8 '14 at 5:09

























        answered Jan 28 '14 at 12:11









        Felix MarinFelix Marin

        67.5k7107141




        67.5k7107141






























            draft saved

            draft discarded




















































            Thanks for contributing an answer to Mathematics Stack Exchange!


            • Please be sure to answer the question. Provide details and share your research!

            But avoid



            • Asking for help, clarification, or responding to other answers.

            • Making statements based on opinion; back them up with references or personal experience.


            Use MathJax to format equations. MathJax reference.


            To learn more, see our tips on writing great answers.




            draft saved


            draft discarded














            StackExchange.ready(
            function () {
            StackExchange.openid.initPostLogin('.new-post-login', 'https%3a%2f%2fmath.stackexchange.com%2fquestions%2f159836%2fint-0-infty-frace-x-sinxx-dx-evaluate-integral%23new-answer', 'question_page');
            }
            );

            Post as a guest















            Required, but never shown





















































            Required, but never shown














            Required, but never shown












            Required, but never shown







            Required, but never shown

































            Required, but never shown














            Required, but never shown












            Required, but never shown







            Required, but never shown







            Popular posts from this blog

            Mario Kart Wii

            What does “Dominus providebit” mean?

            Antonio Litta Visconti Arese